Preguntas sobre el teorema de Carnot

Este artículo sobre el teorema de Carnot establece que

Todas las máquinas térmicas entre dos depósitos de calor son menos eficientes que una máquina térmica de Carnot que opera entre los mismos depósitos.

Sin embargo, solo prueba que ninguna máquina térmica puede ser más eficiente que la máquina térmica de Carnot (usando una prueba que también usa Sal Khan), y prueba que ninguna máquina irreversible es más eficiente que una máquina térmica de Carnot. Establece en la prueba anterior que

Todos los motores reversibles que operan entre los mismos dos depósitos de calor tienen la misma eficiencia.

y en la última prueba

Ningún motor irreversible es más eficiente que el motor de Carnot que opera entre los mismos dos depósitos.

Estas pruebas derivan en el resultado de que el motor de Carnot es el motor con una eficiencia óptima que está de acuerdo con mi libro de texto que se usa para el autoaprendizaje (Resnick y Halliday, décima edición, aunque no he hecho referencia al libro de Callen, que es más avanzado ya que deseo repasar algunas habilidades matemáticas). Sin embargo, tengo dos preguntas basadas en la misma premisa:

Premisa: El artículo afirma que todas las máquinas térmicas son menos eficientes que una máquina de Carnot.

#1: ¿Por qué entonces una máquina irreversible no puede ser tan eficiente como una máquina de Carnot (ya que esto no viola el resultado de la última prueba, que simplemente establece que no puede ser más eficiente ) ?

#2: El artículo solo prueba que todos los motores reversibles tienen la misma eficiencia que el motor de Carnot. Entonces, ¿cómo no se puede construir un diseño diferente para un motor reversible con tal eficiencia (cuál es la justificación de la singularidad del ciclo de Carnot)?

Si Callen se ocupa de esto, se agradecen las citas. Además, se agradece cualquier enlace que incluya referencias a los primeros termodinámicos (Clausius, Gibbs, Maxwell, etc.). Si bien Carnot trabajó bajo la teoría calórica del calor, también se agradecen enlaces a su obra y su razonamiento.

Respuestas (3)

La prueba detrás del límite superior de Carnot planteado sobre la eficiencia de los motores térmicos es más sólida que esto. Las citas que ha pegado se encuentran entre las diversas declaraciones de la segunda ley de la termodinámica. Aquí les esbozaré algunas de las ideas de la demostración, principalmente para mostrar de dónde vienen estas formulaciones (relacionadas con Carnot) del segundo principio. Inevitablemente, repetiré la mayoría de las cosas que probablemente ya sepa, pero se repiten más con fines de discusión. Hacia el final me referiré más de cerca a sus dos preguntas principales.

La principal cuestión planteada por Carnot es básicamente la siguiente: del segundo principio ya sabemos que es imposible construir una máquina térmica que funcione únicamente con un solo baño de calor. Entonces, la pregunta se convirtió en: ¿cuál es la cantidad máxima de trabajo que podemos lograr con un motor térmico que funciona de manera reversible entre dos caminos de calor, con los que puede intercambiar calor?

Ahora, desde un punto de vista puramente esquemático, sabemos que uno de esos motores debería describirse mediante un ciclo termodinámico que maximice el área bajo la curva en el diagrama PV. Entonces Carnot se dispuso a idear un ciclo termodinámico que satisficiera esto. Recuerde que el trabajo útil neto que proporciona el sistema es igual al área encerrada por un ciclo cerrado, por lo que intuitivamente ya tenemos una idea del tipo de expansiones y compresiones que debe realizar el ciclo: es decir, el costo de la compresión se minimiza al comprimiendo en frío (mínimo T ), y la expansión que produce la máxima cantidad de trabajo al expandirse en caliente (mayor T ), de ahí la elección de las dos partes principales reversibles (sin pérdida alguna, sin producción de entropía) isotérmicas de compresión y expansión del ciclo de Carnot. Aquí está el diagrama PV tomado de wikipedia ("Diagrama pV del ciclo de Carnot". Licenciado bajo CC BY-SA 3.0 a través de Commons):ingrese la descripción de la imagen aquí

Un recordatorio rápido de cada paso involucrado junto con el trabajo/calor proporcionado al exterior (signo menos) o recibido (signo más):

  • 1 a 2: expansión isotérmica reversible, T = T 1 , V 1 V 2 , W 1 = R T 1 en V 2 / V 1 y q 1 = R T 1 en V 2 / V 1 , sin cambio de energía interna, q = W

  • 2 a 3: enfriamiento del agente de trabajo a través de una expansión adiabática reversible, energía interna reducida solo a través del trabajo, q 2 = 0 , V 2 V 3 , T 1 T 2 y W 2 = C v ( T 2 T 1 )

  • 3 a 4: compresión isotérmica reversible en frío T = T 2 , V 3 V 4 , W 3 = R T 2 en V 4 / V 3 y q 3 = R T 2 en V 4 / V 3 ,

  • 4 a 1: calentamiento por compresión adiabática reversible: T 2 T 1 , V 4 V 1 , q 4 = 0 y W 4 = C v ( T 1 T 2 )

Todos los ingredientes para calcular la eficiencia térmica η C a r norte o t , dado por el trabajo neto proporcionado por el sistema al medio ambiente dividido por el calor total recibido durante un ciclo (una idea importante es usar las transformaciones adiabáticas para expresar W en términos de T 1 , 2 y V 1 , 2 ). Una vez hecho esto, debería ser una función única de las dos temperaturas del baño de calor (en Kelvin):

η C a r norte o t = T 1 T 2 T 1 = 1 T 2 T 1
Para ver por qué cualquier motor irreversible tendría una eficiencia más baja, reemplace cualquiera de los 4 pasos del ciclo por uno irreversible y vuelva a calcular la eficiencia, por ejemplo, reemplacemos la expansión adiabática de 2 a 3 por un proceso irreversible, para nuestros propósitos un bastará una simple expansión libre (ver Gay-Lussac ), durante la cual no se realiza ningún trabajo y T 1 permanece constante, este proceso es seguido inmediatamente por otro proceso irreversible, correspondiente al intercambio de calor (de ahí la irreversibilidad) con el baño frío tan pronto como se establece el contacto, para llegar a T 2 . Si los otros 3 pasos no se ven afectados (es decir, son reversibles), la eficiencia se convierte en:
η = η C a r norte o t C V ( T 1 T 2 ) R T 1 en V 2 / V 1 < η C a r norte o t
Para convencerte aún más, puedes repetir el cálculo para la sustitución de cualquiera de los otros 3 pasos, por uno irreversible, y siempre encontrarás η < η C a r norte o t . Si lo prefiere, en términos de entropía, se puede demostrar que cualquier proceso irreversible tiene menos flujo de calor hacia el sistema durante una expansión y más flujo de calor fuera del sistema durante una compresión, lo que simplemente significa que se le da más entropía al medio ambiente que la que recibe del sistema. lo que, en consecuencia, transforma el teorema de Clausius en una desigualdad , es decir
d q d T 0

Con respecto a su segunda pregunta, la idea clave es que ningún otro motor puede producir una mayor eficiencia que el de Carnot (que conceptualmente ahora esperamos que sea cierto, recuerde los puntos anteriores sobre cómo construir un ciclo que maximice η ), pero esto no significa que no haya otros motores con transformaciones reversibles que no puedan producir la misma eficiencia, tomemos, por ejemplo, el motor Stirling (un ciclo de 4 pasos nuevamente):

Para el motor Stirling puede demostrar que la eficiencia es:

η S t i r yo i norte gramo = R ( T 1 T 2 ) en V 2 / V 1 R T 1 en V 2 V 1 = 1 T 2 T 1 = η C a r norte o t

lo que debería convencerte de que el ciclo de Carnot no es único. Para resumir, todo esto lleva a otra declaración del segundo principio de la termodinámica: no hay máquina que realice un proceso cíclico con una eficiencia mayor que η C a r norte o t .

Estoy confundido en cuanto a por qué dice que el ciclo de Stirling tiene la eficiencia de Carnot. El ciclo de Carnot es el único ciclo que opera entre sólo dos temperaturas y que tiene la máxima eficiencia. Cualquier otro ciclo, aunque sea reversible , debe tener una eficiencia menor: para que un ciclo que no sea de Carnot sea reversible, debe operar entre más de dos temperaturas (en general, un continuo de temperaturas).
@march Eso no es cierto, el ciclo de Carnot no es único (piense intuitivamente en el hecho de que hay otras formas de llegar a la misma área cerrada del Carnot en el diagrama PV), hay otros ciclos idealizados reversibles que alcanzan la misma eficiencia , de hecho, el cálculo de la eficiencia térmica del motor Stirling es bastante fácil, estoy seguro de que puede convencerse de esto. Lo que hace que Carnot sea único es que los procesos involucrados facilitan la prueba de que tal eficiencia no puede ser superada incluso en otros escenarios idealizados.
@Phonon. En lo que respecta a la misma área encerrada por el diagrama PV, eso solo me dice que el trabajo neto realizado es el mismo aunque permite diferencias en los flujos de calor. La única forma de recuperar la eficiencia de Carnot es si el calor expulsado durante el segundo proceso isocórico es el mismo que el calor absorbido durante el primer proceso isocórico. No pensé que esto fuera posible, así que siempre pensé que el propósito del regenerador era aprovechar la q a través de lo finito Δ T durante los procesos isocóricos. Tendré que pensar en esto.
¿Llegaste a alguna solución?

Parece que te gustaría leer algunos libros clásicos sobre termodinámica; uno especialmente bueno es el de Fermi . Tiene muchos buenos argumentos; breve y simple, pero lógicamente hermético.

Para su primera pregunta: un proceso irreversible aumenta la entropía del universo. Si un motor irreversible tuviera la misma eficiencia que uno reversible, podríamos hacerlo funcionar junto con el reversible al revés, llegando exactamente al mismo estado inicial del universo. Pero aumentamos la entropía del universo haciendo funcionar el motor irreversible, por lo que en algún momento debemos haber disminuido la entropía. Esto es imposible.

Para su segunda pregunta: todos los motores reversibles tienen la misma eficiencia. Todos estos motores se ven prácticamente iguales: solo se le permite transferir calor de los depósitos de forma isotérmica (con el material del motor a la misma temperatura que los depósitos), y se le permite hacer cualquier otra cosa en el medio. Carnot hace adiabáticos en el medio, pero podrías hacer isocoros. O una adiabática en un sentido y una isocórica en el otro. O una isobárica, luego una isocórica, luego una adiabática, etc. Hay muchas posibilidades.

Gracias por la recomendación del libro, así como por las respuestas a las dos preguntas. He oído hablar del libro de Fermi, pero actualmente tengo el libro de Callen. Le daré un vistazo.
Revisé el libro de Fermi y es excelente para mis propósitos, gracias por la recomendación.
@Cicerón. Secundo el libro de Fermi. Es corto y directo, y su derivación de la desigualdad de Clausius es perfecta.
@march Gracias por la información. Aprecio su brevedad y claridad.

El motor reversible es el más eficiente y todos los motores reversibles tienen la misma eficiencia. Se puede mostrar así: si hay una máquina X que es más eficiente que el motor R (reversible), entonces podríamos invertir la R para dar al depósito caliente el calor que tomó X, pero para hacer eso tendríamos que tomar más calor. del depósito frío, entonces X porque X es más eficiente por suposición. En total, el trabajo de X, reducido por el trabajo de R en la inversión, proviene solo de tomar el calor del depósito frío y eso no es posible según la segunda ley. Formulación de Kelvin de la segunda ley: No es posible tener tal cambio de estado de un sistema donde el único efecto sería convertir completamente una cierta cantidad de calor en trabajo. Es una afirmación de que el calor va en una sola dirección y esta es una declaración algo fenomenológica de la segunda ley.